Board logo

标题: 请看GWD-5-15 [打印本页]

作者: lololo    时间: 2004-12-11 09:08     标题: 请看GWD-5-15

Q15:

Lightbox, Inc., owns almost all of the movie theaters in Washington County and has announced plans to double the number of movie screens it has in the county within five years. Yet attendance at Lightbox’s theaters is only just large enough for profitability now and the county’s population is not expected to increase over the next ten years. Clearly, therefore, if there is indeed no increase in population, Lightbox’s new screens are unlikely to prove profitable.

Which of the following, if true about Washington County, most seriously weakens the argument?

Though little change in the size of the population is expected, a pronounced shift toward a younger, more affluent, and more entertainment-oriented population is expected to occur. The sales of snacks and drinks in its movie theaters account for more of Lightbox’s profits than ticket sales do. In selecting the mix of movies shown at its theaters, Lightbox’s policy is to avoid those that appeal to only a small segment of the movie going population. Spending on video purchases, as well as spending on video rentals, is currently no longer increasing. There are no population centers in the county that are not already served by at least one of the movie theaters that Lightbox owns and operates.

The key is A, I chose C. C is also logic, since the company can also increase the population who come to see the cinema.Please give me some tips. Thank you!


作者: wretdgdh    时间: 2004-12-11 09:11

C.这个行为可能之前就已经这样做了。所以不能表示增加之后能够带来额外的收益。
作者: ppp1    时间: 2004-12-11 09:12

C is out of the scope,文中没有提到怎么选电影,如果要weaken evidences即人口十年之内不会增加, A中阐明了潜在人口的增加,起到了weaken的作用. 呵呵..其实我开始和你犯了一样的错误.




欢迎光临 国际顶尖MBA申请交流平台--TOPWAY MBA (http://forum.topway.org/) Powered by Discuz! 7.2